Difference between revisions of "2015 AMC 10B Problems/Problem 2"

(Solution)
m (Solution: LaTeXed "2")
(14 intermediate revisions by 6 users not shown)
Line 1: Line 1:
 
==Problem==
 
==Problem==
  
Marie does three equally time-consuming tasks in a row without taking breaks. She begins the first task at 1:00 PM and finishes the second task at 2:40 PM. When does she finish the third task?
+
Marie does three equally time-consuming tasks in a row without taking breaks. She begins the first task at <math>1\!:\!00</math> PM and finishes the second task at <math>2\!:\!40</math> PM. When does she finish the third task?
  
 
<math> \textbf{(A) }\text{3:10 PM}\qquad\textbf{(B) }\text{3:30 PM}\qquad\textbf{(C) }\text{4:00 PM}\qquad\textbf{(D) }\text{4:10 PM}\qquad\textbf{(E) }\text{4:30 PM} </math>
 
<math> \textbf{(A) }\text{3:10 PM}\qquad\textbf{(B) }\text{3:30 PM}\qquad\textbf{(C) }\text{4:00 PM}\qquad\textbf{(D) }\text{4:10 PM}\qquad\textbf{(E) }\text{4:30 PM} </math>
  
 
==Solution==
 
==Solution==
Marie does her work twice in <math>1</math> hour and <math>40</math> minutes. Therefore, one task should take <math>50</math> minutes to finish. <math>50</math> minutes after <math>2:40</math> PM is <math>3:30</math> PM, so our answer is (B).
+
Marie finishes <math>2</math> tasks in <math>1</math> hour and <math>40</math> minutes. Therefore, one task should take <math>50</math> minutes to finish. <math>50</math> minutes after <math>2\!:\!40</math> PM is <math>3\!:\!30</math> PM, so our answer is <math>\boxed{\textbf{(B) }\text{3:30 PM}}</math>
 +
 
 +
==Video Solution==
 +
https://youtu.be/7CcxOXjv3i8
 +
 
 +
~savannahsolver
 +
 
 +
==See Also==
 +
{{AMC10 box|year=2015|ab=B|num-b=1|num-a=3}}
 +
{{MAA Notice}}

Revision as of 17:04, 13 July 2020

Problem

Marie does three equally time-consuming tasks in a row without taking breaks. She begins the first task at $1\!:\!00$ PM and finishes the second task at $2\!:\!40$ PM. When does she finish the third task?

$\textbf{(A) }\text{3:10 PM}\qquad\textbf{(B) }\text{3:30 PM}\qquad\textbf{(C) }\text{4:00 PM}\qquad\textbf{(D) }\text{4:10 PM}\qquad\textbf{(E) }\text{4:30 PM}$

Solution

Marie finishes $2$ tasks in $1$ hour and $40$ minutes. Therefore, one task should take $50$ minutes to finish. $50$ minutes after $2\!:\!40$ PM is $3\!:\!30$ PM, so our answer is $\boxed{\textbf{(B) }\text{3:30 PM}}$

Video Solution

https://youtu.be/7CcxOXjv3i8

~savannahsolver

See Also

2015 AMC 10B (ProblemsAnswer KeyResources)
Preceded by
Problem 1
Followed by
Problem 3
1 2 3 4 5 6 7 8 9 10 11 12 13 14 15 16 17 18 19 20 21 22 23 24 25
All AMC 10 Problems and Solutions

The problems on this page are copyrighted by the Mathematical Association of America's American Mathematics Competitions. AMC logo.png